Đến nội dung

Hình ảnh

[TOPIC] ÔN THI BẤT ĐẲNG THỨC $\boxed{\text{THPT CHUYÊN}}$ LỚP $10$ năm $2018-2019$

bất đẳng thức holder cosi bunhiacopxki

  • Chủ đề bị khóa Chủ đề bị khóa
Chủ đề này có 318 trả lời

#141
hoangkimca2k2

hoangkimca2k2

    Sĩ quan

  • Thành viên
  • 477 Bài viết

Bài 62

Cho các số thực $a, b, c$ bất kì. Chứng minh:

${\left( {{a^2} + {b^2} + {c^2} - ab - bc - ca} \right)^3} \ge {\left( {a - b} \right)^3}{\left( {b - c} \right)^3} + {\left( {b - c} \right)^3}{\left( {c - a} \right)^3} + {\left( {c - a} \right)^3}{\left( {a - b} \right)^3}$ (Trần Hoàng Nam)

@hoangkimca2k2: bạn xem lại bài 60 cái, cho a=b=c=1 thì VT<VP luôn rồi :D

Đã sửa :D

Bài 64: Cho $x,y,z>0$ thỏa mãn $x+y+z=9$. Tìm giá trị nhỏ nhất của biểu thức $P=\frac{x^{3}+y^{3}}{xy+9}+\frac{y^{3}+z^{3}}{yz+9}+\frac{z^{3}+x^{3}}{9+xz}$


Bài viết đã được chỉnh sửa nội dung bởi hoangkimca2k2: 21-04-2018 - 21:33

  N.D.P 

#142
MoMo123

MoMo123

    Sĩ quan

  • Điều hành viên THCS
  • 334 Bài viết

Bài 52. $a^{4}\,+\, b^{4}\,+\, c^{4}\,=\,3$. Chứng minh rằng: P=$(\frac{a^{3}}{b^{5}}+ \frac{b^{3}}{c^{5}}+ \frac{c^{3}}{a^{5}})\,(\frac{b^{3}}{a^{5}}+ \frac{c^{3}}{b^{5}}+ \frac{a^{3}}{c^{5}})\,\geqq \,9$

Dễ dàng chứng minh được $\sum \frac{a^3}{b^5}  và \sum \frac{b^3}{a^5}$$ \geq \sum \frac{1}{a^2}$ 

$-> P \geq (\sum \frac{1}{a^2})^2\geq (\frac{9}{a^2+b^2+c^2})^2 \geq 9$

Mọi người lưu ý: Bài chưa được làm hãy tô màu đen, còn bài được làm rồi mới tô màu đỏ


Bài viết đã được chỉnh sửa nội dung bởi MoMo123: 21-04-2018 - 19:03


#143
phamhuy1801

phamhuy1801

    Trung sĩ

  • Thành viên
  • 181 Bài viết

Bài $64$: (Tìm giá trị nhỏ nhất)

$ P=\sum \frac{x^3+y^3}{xy+9} = \sum x^3\left (\frac{1}{xy+9}+\frac{1}{xz+9} \right ) $

$ \geq \sum \frac{4x^3}{x(y+z)+18} = \sum \frac{4x^3}{x(9-x)+18} = \sum \frac{4x^3}{-x^2+9x+18} $

Có thể chỉ ra $\frac{4x^3}{-x^2+9x+18} \geq \frac{11x-21}{4} \Leftrightarrow (x-3)^2(9x+14) \geq 0$ bằng phương pháp cân bằng hệ số hoặc phương pháp tiếp tuyến.

Cộng lại các bất đẳng thức tương tự, $P \geq 9$

 

Đóng góp thêm 1 bài toán (made by me từ hồi lớp 9 :3)

 

Bài $65$: Cho các số dương $x,y,z$. Chứng minh: 

                                               $(x+\frac{yz}{x})(y+\frac{zx}{y})(z+\frac{xy}{z}) \ge 4\sqrt[3]{(x^3+y^3)(y^3+z^3)(z^3+x^3)}$


Bài viết đã được chỉnh sửa nội dung bởi phamhuy1801: 21-04-2018 - 21:20


#144
MoMo123

MoMo123

    Sĩ quan

  • Điều hành viên THCS
  • 334 Bài viết

 

$\boxed{\text{Bài 48}} $Cho $a,b,c$ là các số thực không âm, không vượt quá 4 thỏa mãn $a+b+c=6$ , tìm Max và min của biểu thức 

                                                   $A= \sum a^4+24(1-a)(1-b)(1-c)$

Đặt                                                                                                  $a= x+2$

$b=y+2$

$c=z+2$

$\Leftrightarrow x+y+z=0$

$x,y,z\in[-2;2]$

Trong 3 số $x,y,z$ tồn tại ít nhất 2 số cùng dấu, giả sử đó là x,y $\Leftrightarrow xy \geq 0$

Ta có : $x^2+y^2+z^2 \leq (x+y)^2+z^2=2z^2 \leq 8$

$A=(x+2)^4+(y+2)^4+(z+2)^4 -24(x+1)(y+1)(z+1) =x^4+y^4+z^4 +8(x^3+y^3+z^3-3xyz) +24(x^2+y^2+z^2-xy-yz-zx)+24 \geq 24 $

Dấu bằng xảy ra tại $x=y=z=0$

Xét tích $(4-a)(4-b)(4-c) \geq 0$ 

$\Rightarrow ab+bc+ca \geq 8$

$\Rightarrow -24(xy+yz+zx) \leq 96$

Ta chứng minh $ x^4+y^4 +z^4 \leq 4x^2+4y^2+4z^2 \leq 32$

$\Leftrightarrow x^2(4-x^2)+y^2(4-y^2)+z^2(4-z^2) \geq 0$(đúng)

$A= x^4+y^4 +z^4 +24(x^2+y^2+z^2) -24(xy+yz+zx)+24 \leq 32+24.8+96 +24 =344$

Dấu bằng xảy ra <=> $ (a,b,c)=(4;2;0) $ và các hoán vị của nó 


Bài viết đã được chỉnh sửa nội dung bởi MoMo123: 21-04-2018 - 21:29


#145
tr2512

tr2512

    Thượng sĩ

  • Thành viên
  • 272 Bài viết

Bài 60: Cho $a,b,c>0$ và $a+b+c=3$. Chứng minh rằng: $\frac{a}{b+c}+\frac{b}{c+a}+\frac{c}{a+b}\geq \frac{21}{16}+\frac{27(a^{3}+b^{3}+c^{3})}{16(a+b+c)^{3}}$

Theo quan điểm của mình đây là một bài khá chặt :D

Vì thế mình sẽ sử dụng cách hơi tà đạo tí (p,q,r) 

Quy đồng VT bất đẳng thức:

$\frac{a}{{b + c}} + \frac{b}{{a + c}} + \frac{c}{{a + b}} \ge \frac{{21}}{{16}} + \frac{{27\left( {{a^3} + {b^3} + {c^3}} \right)}}{{16{{\left( {a + b + c} \right)}^3}}}\\ \Leftrightarrow \frac{{{a^3} + {b^3} + {c^3} + 3{\rm{a}}bc + ab\left( {a + b} \right) + bc\left( {b + c} \right) + ca\left( {c + a} \right)}}{{\left( {a + b} \right)\left( {b + c} \right)\left( {c + a} \right)}} \ge \frac{{21}}{{16}} + \frac{{{a^3} + {b^3} + {c^3}}}{{16}}$ 

Đặt $ab+bc+ca=q; abc=r$ và sử dụng các biến đổi:

${a^3} + {b^3} + {c^3} = {\left( {a + b + c} \right)^3} - 3\left( {a + b + c} \right)\left( {ab + bc + ca} \right) + 3{\rm{a}}bc = 27 - 9q + 3{\rm{r}}\\ ab\left( {a + b} \right) + bc\left( {b + c} \right) + ca\left( {a + c} \right) = \left( {ab + bc + ca} \right)\left( {a + b + c} \right) - 3{\rm{a}}bc = 3q - 3{\rm{r}}\\ \left( {a + b} \right)\left( {b + c} \right)\left( {c + a} \right) = \left( {a + b + c} \right)\left( {ab + bc + ca} \right) - abc = 3q - r$

Viết lại bất đẳng thức:

$\Rightarrow \frac{{27 - 9q + 3{\rm{r}} + 3{\rm{r}} + 3q - 3{\rm{r}}}}{{3q - r}} \ge \frac{{21}}{{16}} + \frac{{27 - 9q + 3{\rm{r}}}}{{16}}\\ \Leftrightarrow \frac{{27 - 6q + 3{\rm{r}}}}{{3q - r}} \ge \frac{{3{\rm{r}} - 9q + 48}}{{16}}\\ \Leftrightarrow 432 - 96q + 48{\rm{r}} \ge 9q{\rm{r}} - 27{q^2} + 144q - 3{{\rm{r}}^2} + 9q{\rm{r}} - 48{\rm{r}}\\ \Leftrightarrow 3{{\rm{r}}^2} + r\left( {96 - 18q} \right) + 27{q^2} - 240q + 432 \ge 0$

Theo C-S: $q\le3 \Rightarrow 96-18q\ge0$

Áp dụng bất đẳng thức Schur bậc 3 dạng p, q, r: $r\ge{\frac{{4q - 9}}{3}}$

Ta có:$3{{\rm{r}}^2} + r\left( {96 - 18q} \right) + 27{q^2} - 240q + 432 \ge 3{\left( {\frac{{4q - 9}}{3}} \right)^2} + \left( {\frac{{4q - 9}}{3}} \right)\left( {96 - 18q} \right) + 27{q^2} - 240q + 432 = \frac{1}{3}\left( {3 - q} \right)\left( {171 - 25q} \right)\ge0$

Mà bất đẳng thức cuối luôn đúng do $q\le3$.

Hoàn tất chứng minh.


Bài viết đã được chỉnh sửa nội dung bởi tr2512: 21-04-2018 - 21:29


#146
Khoa Linh

Khoa Linh

    Thiếu úy

  • Thành viên
  • 601 Bài viết

Bài 66: Cho các số dương a, b, c thỏa mãn $abc\leq 1$. Chứng minh rằng:

$\frac{a}{b}+\frac{b}{c}+\frac{c}{a}\geq a+b+c$

p/s: Bài không khó nhưng đẹp  :D


$\sqrt[LOVE]{MATH}$

"If I feel unhappy, I do mathematics to become happy. If I am happy, I

 

do mathematics to keep happy" - Alfréd nyi 


#147
hoangkimca2k2

hoangkimca2k2

    Sĩ quan

  • Thành viên
  • 477 Bài viết

Bài $64$: (Tìm giá trị nhỏ nhất)

$ P=\sum \frac{x^3+y^3}{xy+9} = \sum x^3\left (\frac{1}{xy+9}+\frac{1}{xz+9} \right ) $

$ \geq \sum \frac{4x^3}{x(y+z)+18} = \sum \frac{4x^3}{x(9-x)+18} = \sum \frac{4x^3}{-x^2+9x+18} $

Có thể chỉ ra $\frac{4x^3}{-x^2+9x+18} \geq \frac{11x-21}{4} \Leftrightarrow (x-3)^2(9x+14) \geq 0$ bằng phương pháp cân bằng hệ số hoặc phương pháp tiếp tuyến.

Cộng lại các bất đẳng thức tương tự, $P \geq 9$

 

 

Cảm ơn bạn, mình đã sửa.

Cách của bạn rất hay sau đây mình xin trình bày thêm $1$ cách khác:

Cách 2: 

Theo BDT $Holder$ ta có: 

$(x^{\frac{3}{2}}+y^{\frac{3}{2}}+z^{\frac{3}{2}})^{\frac{2}{3}}(1+1+1)^{\frac{1}{3}}\geq x+y+z$

$\Rightarrow 3(x^{\frac{3}{2}}+y^{\frac{3}{2}}+z^{\frac{3}{2}})^{2}\geq (x+y+z)^{3}$

Lại có theo BDT $Cauchy-schwarz$: 

$\frac{x^{3}}{xy+9}+\frac{y^{3}}{yz+9}+\frac{z^{3}}{9+xz}\geq \frac{(x^{\frac{3}{2}}+y^{\frac{3}{2}}+z^{\frac{3}{2}})^{2}}{xy+yz+xz+27}\geq \frac{(x+y+z)^{3}}{3(27+xy+yz+xz)}$

Tương tự rồi cộng vế theo vế ta có:$P\geq \frac{2(x+y+z)^{3}}{3(27+xy+yz+xz)}$

Mặt khác theo BDT $AM-GM$: ta có $xy+yz+xz\leq \frac{(x+y+z)^{2}}{3}$. Đến đây thì dễ rồi :D


  N.D.P 

#148
HelpMeImDying

HelpMeImDying

    Trung sĩ

  • Thành viên
  • 108 Bài viết

Bài 66: Cho các số dương a, b, c thỏa mãn $abc\leq 1$. Chứng minh rằng:

$\frac{a}{b}+\frac{b}{c}+\frac{c}{a}\geq a+b+c$

p/s: Bài không khó nhưng đẹp  :D

Chia cả 2 vế cho abc

bđt trở thành: $\frac{1}{b^{2}c}+\frac{1}{c^{2}a}+\frac{1}{a^{2}b}\geq \frac{1}{ab}+\frac{1}{bc}+\frac{1}{ca}$

Có: $\frac{1}{a^{2}b}+\frac{1}{a^{2}b}+\frac{1}{c^{2}a}\geq \frac{3}{\sqrt[3]{a^{5}b^{2}c^{2}}}\geq \frac{3}{a}$

Tương tự ta được BĐT cần chứng minh



#149
hoangkimca2k2

hoangkimca2k2

    Sĩ quan

  • Thành viên
  • 477 Bài viết

Bài 67: Cho $a,b,c>0$. Chứng minh rằng $\sqrt{(a^{2}b+b^{2}c+c^{2}a)(ab^{2}+bc^{2}+ca^{2})}\geq abc+\sqrt[3]{(a^{3}+abc)(b^{3}+abc)(c^{3}+abc)}$


Bài viết đã được chỉnh sửa nội dung bởi MoMo123: 21-04-2018 - 22:24

  N.D.P 

#150
hoangkimca2k2

hoangkimca2k2

    Sĩ quan

  • Thành viên
  • 477 Bài viết

Bài 68: Cho $a,b,c>0$. Chứng minh rằng $(a+b+c)(\frac{1}{a}+\frac{1}{b}+\frac{1}{c})\geq 3+3\sqrt[3]{\frac{3(a+b+c)(a+b)(b+c)(c+a)}{(ab+bc+ac)^{2}}}$


  N.D.P 

#151
hoangkimca2k2

hoangkimca2k2

    Sĩ quan

  • Thành viên
  • 477 Bài viết

Bài 69: Cho $a,b,c>0$ và $abc=1$. Chứng minh rằng $\sqrt{a^{2}-a+1}+\sqrt{b^{2}-b+1}+\sqrt{c^{2}-c+1}\geq a+b+c$


  N.D.P 

#152
DOTOANNANG

DOTOANNANG

    Đại úy

  • ĐHV Toán Cao cấp
  • 1609 Bài viết

Bài 52. $a^{4}\,+\, b^{4}\,+\, c^{4}\,=\,3$. Chứng minh rằng: $(\frac{a^{3}}{b^{5}}+ \frac{b^{3}}{c^{5}}+ \frac{c^{3}}{a^{5}})\,(\frac{b^{3}}{a^{5}}+ \frac{c^{3}}{b^{5}}+ \frac{a^{3}}{c^{5}})\,\geqq \,9$

 

Chỉ đơn giản là $VT^{2}\,\geqq \, \frac{9}{\sqrt[3]{a^{4}\,b^{4}\,c^{4}}}\,\geqq\, \frac{9}{\frac{a^{4}+ b^{4}+ c^{4}}{3}}\,= \,9$

 

 


Bài viết đã được chỉnh sửa nội dung bởi DOTOANNANG: 23-04-2018 - 08:51


#153
DOTOANNANG

DOTOANNANG

    Đại úy

  • ĐHV Toán Cao cấp
  • 1609 Bài viết

45. Cho các số dương $a,\,, b\,,\, c$ thỏa $a^{2}+ b^{2}- ab\,=\, c^{2}$. Chứng minh rằng: $(a-c)\,(b-c)\,\leq \,0$

 

Ta có: $a^{2}\,+\, b^{2}\,-\, ab\,= \,c^{2}\,\Leftrightarrow \, a^{2}\,+ \,b^{2}\,-\, 2\,ab.\,\cos 60^{0} \,= \,c^{2}$ (theo định lí hàm cosine)

 

nên $c$ là cạnh đối diện góc $60^{0}$ và $c= mid\left \{ a, b, c \right \}$ (quan hệ giữa cạnh và góc đối diện). Do đó: $(a-c)\,(b-c)\,\leq \,0$

 


#154
DOTOANNANG

DOTOANNANG

    Đại úy

  • ĐHV Toán Cao cấp
  • 1609 Bài viết

Bài 53. Cho $a\,,\,b\,,\,c$ là 3 cạnh của 1 tam giác. Chứng minh rằng: $\frac{a^{3}}{b}\,+\, \frac{b^{3}}{c}\,+ \,\frac{c^{3}}{a}\,\geqq \,\frac{3\,(a^{3}\,+\, b^{3}\,+\, c^{3})}{a\,+\, b\,+\, c}$

 

$(a+ b+ c)\,(\frac{a^{3}}{b}+ \frac{b^{3}}{c}+ \frac{c^{3}}{a})$

 

$=\,(a^{3}+ b^{3}+ c^{3})\,+ \,(\frac{a^{4}}{b}+ \frac{b^{4}}{c}+ \frac{c^{4}}{a})\,+\, (\frac{ab^{3}}{c}+ \frac{bc^{3}}{a}+ \frac{ca^{3}}{b})$

 

$\geqq \,(a^{3}+ b^{3}+ c^{3})\,+ \,(\frac{a^{4}}{b}+ \frac{b^{4}}{c}+ \frac{c^{4}}{a})\,+\, (a^{2}b+ b^{2}c+ c^{2}a)$

 

$= \,(a^{3}+ b^{3}+ c^{3})\,+ \,(\frac{a^{4}}{b}+ a^{2}b)\,+\, (\frac{b^{4}}{c}+ b^{2}c)\,+\, (\frac{c^{4}}{a}+ c^{2}a)$

 

$\geqq \,3\,(a^{3}+ b^{3}+ c^{3})$

 

Đây là topic ôn thi chuyên mà có hiện tượng sai sót này. Mình thật sự rất buồn cho những ai đang quan tâm đến topic này và mình hứa sẽ cố gắng hơn để hỗ trợ các em hoàn thành kì thi thật tốt. Xin lỗi tất cả anh em!

 


Bài viết đã được chỉnh sửa nội dung bởi DOTOANNANG: 23-04-2018 - 08:49


#155
DOTOANNANG

DOTOANNANG

    Đại úy

  • ĐHV Toán Cao cấp
  • 1609 Bài viết

Bài 54. Cho các số dương $a\,,\,b\,,\,c$. Chứng minh rằng: $\frac{a}{b}\,+ \,\frac{b}{c}\,+\, \frac{c}{a}\,+\, 1\,\geqq \,\frac{2\sqrt{2}}{3}\,(\sqrt{\frac{b+c}{a}}\,+\, \sqrt{\frac{c+a}{b}}\,+\, \sqrt{\frac{a+ b}{c}})$

 

Hướng dẫn ngắn: $\frac{b+ c}{c}+ \frac{2\,c}{a}\geqq 2\,\sqrt{2} \,\,\sqrt{\frac{b+ c}{a}}$



#156
DOTOANNANG

DOTOANNANG

    Đại úy

  • ĐHV Toán Cao cấp
  • 1609 Bài viết

Bài 58

Cho các số thực không âm $a\,,\, b\,, \,c$ thỏa mãn $a+b+c\,=\,3$. Chứng minh:

$$\frac{a}{{\sqrt {2a + b} }} \,+ \,\frac{b}{{\sqrt {2b + c} }} \,+ \,\frac{c}{{\sqrt {2c + a} }}\, \le\, \sqrt 3$$

 

Ta có: $\left [ \sum_{cycl}^{ }\left (\, \sqrt{a}\, \,\sqrt{\frac{a}{2\,a+ b}} \,\right )^{2} \right ]\,\leqq \,\sum_{cycl}^{ }a\, \sum_{cycl}^{ }\frac{a}{2\,a+ b}$

 

Hiển nhiên quá! Ta biết là phải đi chứng minh: $S\,=\,\frac{a}{2a+ b}\,+\, \frac{b}{2b+ c}\,+\, \frac{c}{2c+ a}\,\leqq\, 1$

 

Đặt: $\frac{b}{a}\,, \,\frac{c}{b}\,,\, \frac{a}{c}\,=\, u\,,\, v\,,\, w$

 

hay: $S\, = \, \frac{1}{2+ u}\, + \, \frac{1}{2+ v}\, + \, \frac{1}{2+ w}\, \leqq\,  1$

 

hay: $\frac{12\,+ \,4\,(u+v+w)\,+\, (uv+vw+ wu)}{8\,+ \,4\,(u+v+w)\,+ \,2\,(uv+vw+ wu)\,+\, 1}\,\leqq \, 1$

 

hay: $3\,\leqq\,uv\,+\,vw\,+\,wu$

 

Khỏi cần chứng minh nữa, điều này quá đúng!

 

 

 

 



#157
tr2512

tr2512

    Thượng sĩ

  • Thành viên
  • 272 Bài viết

Bài 62

Cho các số thực $a, b, c$ bất kì. Chứng minh:

${\left( {{a^2} + {b^2} + {c^2} - ab - bc - ca} \right)^3} \ge {\left( {a - b} \right)^3}{\left( {b - c} \right)^3} + {\left( {b - c} \right)^3}{\left( {c - a} \right)^3} + {\left( {c - a} \right)^3}{\left( {a - b} \right)^3}$ (Trần Hoàng Nam)

@hoangkimca2k2: bạn xem lại bài 60 cái, cho a=b=c=1 thì VT<VP luôn rồi :D

Đặt $ x=a-b; y=b-c; z=c-a $ ta thu được $ x+y+z=0 $. Viết lại bất đẳng thức
$ (a^2+b^2+c^2-ab-bc-ca)^3 \ge (a-b)^3.(b-c)^3+(b-c)^3.(c-a)^3+(c-a)^3.(a-b)^3 $
$ \Leftrightarrow  [(a-b)^2+(b-c)^2+(c-a)^2]^3 \ge 8(a-b)^3.(b-c)^3+8(b-c)^3.(c-a)^3+8(c-a)^3.(a-b)^3 $
$ \Leftrightarrow (x^2+y^2+z^2)^3 \ge x^3y^3+y^3z^3+z^3x^3 $
$ \Leftrightarrow (x^2+y^2+z^2)^3 \ge 8(xy+yz+xz)^3 +24x^2y^2z^2 $
$ \Leftrightarrow (x^2+y^2+z^2)^3 \ge (-x^2-y^2-z^2-2xy-2yz-2zx+2xy+2yz+2zx)^3 + 24x^2y^2z^2 $
$ \Leftrightarrow (x^2+y^2+z^2)^3 \ge -(x^2+y^2+z^2)^3 +24x^2y^2z^2 $
$ \Leftrightarrow (x^2+y^2+z^2)^3 \ge 27 x^2y^2z^2 $
Theo bất đẳng thức AM-GM: $ (x^2+y^2+z^2)^3 \ge 27x^2y^2z^2 \ge 12x^2y^2z^2 $
Hoàn tất chứng minh. 

Bài viết đã được chỉnh sửa nội dung bởi tr2512: 22-04-2018 - 13:40


#158
tr2512

tr2512

    Thượng sĩ

  • Thành viên
  • 272 Bài viết

Bài 63:

Cho $a,b,c$ là các số thực không âm thỏa mãn $a+b+c=1$. Chứng minh:

$a\sqrt {a + 2b} + b\sqrt {b + 2c} + c\sqrt {c + 2{\rm{a}}} \le 1$ Dấu bằng xảy ra khi nào ?

P/s: nhớ chỉ ra kĩ dấu bằng :D

Áp dụng bất đẳng thức C-S có:
$ a\sqrt{a+2b}+b\sqrt{b+2c}+c\sqrt{c+2a} = \sqrt{a}.\sqrt{a^2+2ab}+\sqrt{b}.\sqrt{b^2+2bc} + \sqrt{c}.\sqrt{c^2+2ac} \le \sqrt{(a+b+c)(a^2+b^2+c^2+2ab+2bc+2ca)} = \sqrt{(a+b+c)^3} = 1 $
Bất đẳng thức xảy ra khi $a = b = c = \frac{1}{3} $ hoặc $ a = b =0, c = 1 $ và các hoán vị


#159
thanhdatqv2003

thanhdatqv2003

    Trung sĩ

  • Thành viên
  • 159 Bài viết

Cho a,b,c là 3 cạnh của tam giác

Bài 70: C/m: $P^2\geq 12\sqrt{3}S$        (với p là nửa chu vi, S là diện tích)

 

Bài 71: C/m :  $a^3+b^3+c^3 \geq \frac{P^3}{9}$      (với P là chu vi)

 

Bài 72: C/m:  $a^3+b^3+c^3 \geq \frac{4\sqrt{3}}{3}SP$


Bài viết đã được chỉnh sửa nội dung bởi thanhdatqv2003: 23-04-2018 - 16:30

:ohmy: [Không tồn tại các nghiệm nguyên khác không x, y, và z thoả mãn xn + yn = zn trong đó n là một số nguyên lớn hơn 2.  (FERMAT)  :ohmy: 

 

 

 

 


#160
thanhdatqv2003

thanhdatqv2003

    Trung sĩ

  • Thành viên
  • 159 Bài viết

 

Bài 52. $a^{4}\,+\, b^{4}\,+\, c^{4}\,=\,3$. Chứng minh rằng: $(\frac{a^{3}}{b^{5}}+ \frac{b^{3}}{c^{5}}+ \frac{c^{3}}{a^{5}})\,(\frac{b^{3}}{a^{5}}+ \frac{c^{3}}{b^{5}}+ \frac{a^{3}}{c^{5}})\,\geqq \,9$

 

Chỉ đơn giản là $VT^{2}\,\geqq \, \frac{9}{\sqrt[4]{abc}}\,\geqq\, \frac{9}{\frac{a^{4}+ b^{4}+ c^{4}}{3}}\,= \,9$

 

tại sao VT^2 mà =9 và kèm theo AM-GM thì dưới mẫu phải là $\sqrt[3]{a^4b^4c^4}$  ???


:ohmy: [Không tồn tại các nghiệm nguyên khác không x, y, và z thoả mãn xn + yn = zn trong đó n là một số nguyên lớn hơn 2.  (FERMAT)  :ohmy: 

 

 

 

 






Được gắn nhãn với một hoặc nhiều trong số những từ khóa sau: bất đẳng thức, holder, cosi, bunhiacopxki

2 người đang xem chủ đề

0 thành viên, 2 khách, 0 thành viên ẩn danh